The Fast Track - Fall 2014

Page 1

Welcome to Vegas!

The Fast Track ANNUAL RESIDENCY FAIR 2014

Are you ready for fall conference? Las Vegas 2014

Get set for the annual residency fair, incredible lectures and labs, and amazing evening events!

STEMI Alert!

Three STEMI alerts in three shifts, each one a little different!

How does your CV look?

Get the advice of the pro’s on your CV and your personal statements

Using Adderall

A thought-provoking question on the use of performance-enhancing drugs in medical schools


Letter from the Editor

The Track The Fast Fast Track Editors-in-Chief Fall 2014

Giles Gifford, OMS-III Tanner Gronowski, DO Drew Kalnow, DO Jeremy Lacocque, OMS-IV

Editors

Andy Little, DO Danielle Turrin, DO Erin Sernoffsky RC Board Members

Steve Brandon, DO President Andy Little, DO Vice President Andrew Pacitti, DO Treasurer Danielle Turrin, DO Secretary Brian Ault, DO Stephanie D’Agostino, DO Denzil Frost, DO Tanner Grownowski, DO Drew Kalnow, DO Jeremy Lott, DO Jessica Schaberl, DO Joe Sorber, DO Megan Koenig, DO Past President

SC Board Members Kaitlin Fries Judd Shektin Brent Arnold Nick Bair Robert Culley Giles Gifford Angela Kuehn Jeremy Lacocque Cameron Meyer Tiffany Pham Sasha Rihter Ashley Guthrie, DO

President Vice President Treasurer Secretary

I

t’s getting real. This May, I, and thousands of my colleagues, will be physicians.

We meet this reality with both excitement and anxiety. The interns I work with are no longer distant “doctors” to me, they’re me and my classmates – in nine months. After an almost decadelong journey of scientific and medical education, we’re on the cusp of being physicians. Amidst this journey, I repolished my love for journalism and further assimilated with the ACOEP family by working as the Publications and EMS Committee Co-Chair this year. Having the opportunity to read, write, and facilitate the sharing of our experiences through The Fast Track brings a feeling of invaluable solidarity with those in my desired profession. It allows me to bond with people from all around the country, all interested in osteopathic emergency medicine, and calms the emotional experience of our professional journey. I feel so lucky to have found my professional niche so early on in my career. The combination of osteopathic medicine, emergency medicine, EMS, journalism and the warm, family feeling of the ACOEP make me feel like I have really found my professional home, and a perfect mix of my interests. I hope to further define my niche and become more involved in my profession. I hope to continue working with more of you so we can contribute to our profession, grow, and learn from each other as we have the past few years. I sincerely hope you have enjoyed The Fast Track and I wish you all, regardless of your stage in training, an exciting future. We’ve all worked hard for it.

Past President

Printing of this issue sponsored by:

page Cover photo courtesy of Tanner Gronowski

Jeremy Lacocque, OMS-IV Chicago College of Osteopathic Medicine

Interested in contributing? Let us know: FastTrack@ACOEP.org


CONTENTS Presidential Messages................................04 Rosh Review...................................................06

10

STEMI ALERT! by Drew Kalnow, DO

Tricks of the Trade.......................................07 CV’s and Personal Statements...............08 Medical Apps for Medical Students ...16 Hyperlipasemia.............................................18 Perspective From Above..........................20

20

GHANA AEROMEDICINE by Davis, DO by Frederick Sasha Rihter, OMS-III

Dealing with Death.....................................22 Ghana.................................................................24 Brain Doping...................................................26 Pimpology........................................................28

24

GHANA by Frederick Davis, DO

36

RESIDENCY SPOTLIGHT St. Joseph

How to Stand Out in a Good Way.......30 Making the Most of an Interview.........32 View From the Bottom..............................34 Residency Spotlight....................................36 Toxic Algal Blooms.................................38


The Fast Track

PRESIDENTIAL MESSAGE – Fall 2014

Resident Chapter

I

came into my involvement with the ACOEP somewhat by chance. During my intern year my program director needed someone from our program to present in the Case Presentation Competition at Fall Conference that year. I happened to be the most enthusiastic resident that year, so I got to do it. That same year a friend told me about the position he had as a resident chapter representative on the ACOEP Resident Chapter Board. The position sounded like a good way to get involved, so I went for that position as well. I have been lucky enough to be involved ever since. My point here is that is does not have to be difficult to get involved in your national specialty organizations. Organizations like the ACOEP look out for you and fight for your specialty. They give us one, strong, collective voice that can help to get things accomplished. For example, we have been very involved nationally with the upcoming unified GME accreditation process. The ACOEP has had leaders from the organization involved in every meeting and conference call that has occurred thus far, serving as our voice in the process and looking out for our colleagues. Each of you as emergency physicians represent the lifeblood of the ACOEP, without you the organization has no purpose. In my last message as the Resident Chapter president, I want to take the opportunity to encourage each of you to get involved at the national level. I do not want to limit this; however, to the ACOEP. In emergency medicine, we have a number of organizations that represent us. In addition to ACOEP, there is ACEP, SAEM and AAEM, among others. Each organization has their specific purpose, but each serves to better the profession. Find where you fit in, and get involved. This does not have to be a full time job or even require a huge sacrifice. These organizations all have committees and different ways to be involved for those who are interested. Don’t worry if you have not been involved as a student or throughout your career, there is space for all who are interested. The ACOEP Resident Chapter holds our annual elections at our Fall Conference in Las Vegas this October. We are seeking eager, motivated people. If you are interested, more information can be found in the resident section of the ACOEP website. Sincerely, Steve Brandon, DO St. Mary Mercy Hospital

page 4


The Fast Track

Student Chapter

Fall 2014

PRESIDENTIAL MESSAGE –

W

elcome to the Fall edition of the Fast Track! The beginning of another school year marks an exciting time for all of us. First years have jumped in headfirst, motivated to find out what medical school is all about. Second years have made it through year one and are now looking towards the board exam. Third years have completed step 1, finally leaving the classroom to start clinical rotations. Fourth years have begun planning their future and are about to begin the interview process. No matter how far along you may be in your medical school journey, your ACOEP-SC has something to offer you! What better way to increase your love for emergency medicine then spending three days with us at Fall Conference? Your Student Chapter Board has pulled out all the stops to provide you with an educational, yet fun, experience. Conference highlights include the annual residency exposition featuring more than 30 residency programs. Other events include skill labs, rapid-fire lectures, and evening social events. Nowhere else will you have the opportunity to meet so many residents, attendings, and program directors from across the country all in the same place. Mock interviews are back for our fourth year students to practice and receive feedback prior to the residency interview trail. As if all of that wasn’t enough to convince you… did I mention this is all taking place in Las Vegas? With a new school year also comes a change in leadership. It is time for your current Student Chapter Board to hand over the reigns. We are looking for motivated individuals who are passionate in sharing the field of emergency medicine with other students. If you are at all interested in becoming more involved please attend the election informational meeting held at Conference. We hope to see you all in Vegas! Sincerely, Kaitlin Fries, OMS-IV OU-HCOM

page 5


The Fast Track

Fall 2014

Emergency Medicine Review with 1. A 41-year-old woman presents with a severe headache that started 10 hours ago. She has never had a headache this intense before. The headache is associated with photophobia and nausea. What management is indicated? A. Non-contrast head CT and lumbar puncture B. Non-contrast head CT and discharge if normal C. Outpatient MRI D. Symptomatic headache treatment and follow up with neurology if headache resolves

Find more questions like these by visiting roshreview.com

2. A 16-year-old boy presents with headache, nausea and vomiting after a fall. The patient fell from a tree with a brief loss of consciousness. A minute later he was awake and only complained of a mild headache but he has since worsened. A non-contrast CT scan is shown. What management is indicated?

page 6

A. Admit for repeat non-contrast head CT B. Discharge with follow-up with neurology C. Emergent neurosurgical consultation D. Obtain MRI of the brain 3. For which etiology of cardiac arrest does induced hypothermia carry the best improvement in survival with good neurologic outcome? A. Hyperkalemia B. Pulmonary embolism C. Pulseless electrical activity D. Ventricular fibrillation 4. A 73-year-old woman with an indwelling Foley catheter is sent for evaluation from the nursing home because of fever. Which of the following is a criterion for systemic inflammatory response syndrome (SIRS)? A. Heart rate 86 B. Lactic acid 4 mg/dL C. Respiratory rate 22 D. Temperature 37.9째C

Find your Rosh Review Answers on page 42


Tricks of the Trade By Chase Ungs, DO @ Doctors Hospital

Fall 2014

The Fast Track

Fluorescein Staining Complete eye examinations are considered a basic procedure for an EM physician. The quality of the exam is often limited by preparation and set up, which if done correctly can lead to a quick diagnosis. Evaluating the cornea is essential to a complete exam and can be difficult to do when using the fluorescein strips. Here’s a quick and easy way to stain the eye that is easily tolerated by patients: 1. Most patients complain of eye pain and will require some sort of anesthetic. Place 1-2 drops of your preferred anesthetic (i.e. proparacaine, tetracaine, etc) into one or both eyes 2. Obtain fluorescein strip 3. Pull off the plunger of a 10ml saline flush from your IV cart 4. Drop the entire fluorescein strip into the syringe 5. Replace the plunger and prime the syringe into a towel or sink 6. Have the patient tilt their head up, hold the top and bottom eyelids open with one hand and place 2-3 drops from the saline flush in the preferred eye 7. Complete exam as usual with Wood’s lamp, blue filter on the slit lamp, or pen light with a blue filter to exam the cornea

page 7


The Fast Track

Fall 2014

CV’s and Personal Statements By Leigh Hylkema

Doctors Hospital EM Program Coordinator Chair, EM Association of Residency Coordinators

By StorytellERdoc

PERSONAL STATEMENT The personal statement is the portion of the residency application in which you are 100% in control of the content. It is the only part of the application that is unique to you and does not follow a predefined format. The purpose of the personal statement is: 1. To introduce yourself to the program 2. To state your case: Who are you? What brought you to this point? How have you overcome flaws? Why are you a good candidate? What sets you apart from all the other applicants? There is no correct way to write a personal statement, however, the most effective personal statements are honest, direct, and tell a story. The opening paragraph is the most important. It is the introduction that sets the tone and the framework for the rest of the essay. First impressions do matter. The introductory paragraph should grab the reader’s attention and make them want to read more. In the body of the personal statement you should discuss the details of what brought you to this point in your life. Discuss the history, people, or events that influenced you. Discuss what obstacles you overcame and how. Did you have some sort of epiphany that led you to this point? Discuss what led you to your choice of medical specialty to pursue. Discuss what you can bring to the residency program – what sets you apart from other candidates. Personal characteristics that make you a standout applicant Skills Discuss your career goals. Discuss what you want to get out of residency training.

page 8

The body of your personal statement is also the place where you can explain or justify issues in your application that may be considered deficits or weaknesses.

For example: Why did you take time off or repeat part of medical school? Why do you have a low board score, class rank, or rotation grade on your transcript? Why do you have a negative comment in your dean’s letter or letter of recommendation? The personal statement allows you to discuss how you have overcome the obstacles in your life and gives you the opportunity to turn a negative into a positive. The conclusion of your personal statement should reaffirm your interest in the specialty and the program to which you are applying. This should not be a restatement of your curriculum vitae in paragraph form. It is your opportunity to tell the reader something about yourself that cannot be found anywhere else in the application. Avoid redundancy, avoid clichés, and – most of all – do not define what the specialty is for which you are applying. Avoid terms such as: · Emergency Medicine is…. · Emergency Medicine is like… Your reader, presumably, is a practitioner of the specialty and therefore does not need a definition. The reader wants to learn about something they do not know, they want to learn about you. Your personal statement should be honest. Do not be overly profound, clever, or funny. Do not write fiction or fantasy. Be yourself. A good personal statement is truthful, specific, and tells a story. The Electronic Residency Application Service (ERAS) limits the personal statement to 28,000 characters or approximately eight pages. It is recommended, however, that you keep your personal statement of no more than a page and a half. Very few writers can hold the reader’s attention for little more than a page. It is also recommended that you create your personal statement in a plain text format. Do not use special characters or formatting such as bold, italic, or underlining. This will not translate into the ERAS software and may result in the insertion of unwanted characters into your essay. ERAS allows you to create more than one personal statement and to give each one a distinctive, descriptive name. Only one personal statement can be assigned to a program. Programs do not see the names you assign to your personal statement. This allows you to tailor your personal statement to the program to which you are ap-


plying such as including ties to the program, institution, or location. Proofread your personal statement before submitting it to ERAS. Have others who can be objective proofread it as well to spot issues that you overlooked such as misspellings or omitted words. Your proofreader can also help you with the tone of your personal statement to assure that you do not come across as offensive or too humorous. Remember, the personal statement is just that – personal. It is your story. Use it to your advantage.

CURRICULUM VITAE As the saying goes, you have just one chance to make a good first impression. Often the first impression in your professional life is your curriculum vitae (CV). The purpose of a CV is to: · Introduce or market yourself · Supplement an application · Obtain an interview · Apply for awards, scholarships, or grants · Be considered for a promotion · Provide biographical information Your CV is a summary of your academic and professional history and job qualifications. A CV is different from a resume in that it is more extensive and encompassing. The term curriculum vitae comes from the Latin curriculum meaning “course” and vita meaning “life.” A resume, on the other hand, is brief, focused, and seldom more than two pages. The components of a CV are: · Name and contact information: Your name should be at the top of the first page of your CV. It should be in bold and in a larger font than the rest of the document. Your name and page number should be in the top right corner of each succeeding page. · Residency: Include program name, institution, location, and dates. You should also include institution designations such as emergency department volume, trauma center, chest pain center, stroke center, etc. · Education: Include all degrees, institutions, locations, and dates. Indicate if you graduated with honors. · Licensure: Each state license should be listed as a separate entry. · Certifications: You should include all relevant courses, certifications, conferences, or additional training. · Honors: Include all honors, awards, and scholarships. If the title of the honor is not self-evident of why the honor was bestowed, include a descriptor. · Appointments: You should include clinical, academic, administrative, and political experiences. Include committee work, special projects, policy development, or focus studies/

groups. Elaborate on relevant experiences that are not self-explanatory. · Research: Include the principal investigator/supervisor/advisor, topic or title, institution/organization or department, dates, and your title and/or role in the project. · Publications: You should include all published work, use bibliographic citations, and use appropriate notations such as “in press” if accepted for publication or “submitted” if received by the publisher. · Presentations: Include all major or relevant presentations including posters, papers, and lectures and include the title, conference, location, and date. · Scholarly activities: Include competitions, curriculum development, question writing, or other activities of a scholarly nature that do not fit into other CV categories. · Professional society memberships: You should include all society and club memberships and the active dates. · Committee involvement: You should include the committee name/purpose, institution, location, and level of involvement. · Volunteer activities/community service: Include location and dates. Use descriptors if not selfexplanatory. · Optional: Items that you may choose to include are hobbies/interests, marital status, children, citizenship, or visa status

Fall 2014

The Fast Track

Formatting your CV: · Your CV should be printed on plain white paper with one inch margins all around. · Use consistent formatting throughout including fonts, spacing and punctuation. Use a clean, basic font. Bold section headings and double space between sections. Do not use underlining. Each page should be visually balanced and easy for the reader to scan. · Do not use a standardized word processor resume template in that these are not designed for a comprehensive CV format. Block, bulleted entries are the easiest to read and visually pleasing. · Your name should be at the top of every page. · Entries should be listed in descending chronological order – current to the past. · You should include page numbers and a revision date. · Do not include pictures or graphics. · Avoid abbreviations or acronyms. · Your CV should be comprehensive, yet void of any redundant information. The exception to this rule is scholarly projects that can fall in multiple categories such as a research study that became a paper, a poster, abstract, or presentation. You should keep an electronic version of your CV. The file should be labeled “Last Name, First Name – CV.” Should you ever email your CV to someone, this format will save your recipient time in cataloging your document.

page 9


The Fast Track

Fall 2014

STEMI ALERT! By Drew Kalnow, DO Doctors Hospital On a recent three shift stretch in the ED, I was involved in three STEMI alerts, all with very different outcomes. Each case is described in this article. CASE 1: (2330) 55yo male presents to the ED via EMS complaining of severe, crushing chest pain with associated dyspnea, diaphoresis and nausea. He appears uncomfortable and anxious, stating “I’m having another MI!”. His symptoms started approximately 40 minutes prior to arrival, when he noted sudden, central chest pain that radiated down his left arm. He took four baby aspirin and 3 nitroglycerin tabs with minimal relief prior to arrival to the ED. PMH: HTN, HLD, CAD (2 previous MIs and 4 stents) PSHX: Coronary artery stent placement Social Hx: smokes 1-2 ppd, no EtOH, married Exam: Vitals- BP 84/55, P 67, R 18, spO2 95%, pain 10/10

Gen- AOx3, anxious, in distress

Pulm: CTA, no acute distress

Cardiac: RRR, weak but symmetric pulses

ABD: soft, non-tender, non-distended

Initial ED EKG: (2332)

page 10


ED Course:

The Fast Track

Initial testing: CXR: non- acute chest CBC: 18/16.6/44/133 CMP: 140/3.3/101/22/9/1.2/170 Trop T: <0.010 CPK: 55 (low, no CKMB differentiated) proBNP: 603 VBG: 7.32/38.8

Fall 2014

After resident evaluation of the patient, EKG (and previous EKG from 1998) and consultation with the attending physician, a STEMI alert was called at 2347.

2355: patient in cath lab 2358: Heparin 4000u given 0011: Cardiologist arrives in Cath Lab Hospital Course: Patient became increasingly anxious during cath procedure to the point of uncontrolled agitation despite sedation with Versed. During the procedure, the ED was called to return to the Cath Lab in order to paralyze and intubate the patient. The patient was found to have triple vessel disease with disease of his branching vessels. A stent was placed in the LAD and an intra-aortic balloon pump was placed as a temporizing measure and the cardiovascular surgeon was notified of the need for immediate open heart surgery. Patient received emergent salvage triple vessel bypass, he was able to be weaned off of cardiopulmonary bypass with the support of high dose pressors and the balloon pump. In the recovery suite, the patient developed v-fib arrest while the cardiovascular surgeon and intensivist were present. ACLS was immediately initiated and, since the chest was only closed with a temporary membrane, the chest was opened and cardiac massage was initiated. The patient again converted to a normal sinus rhythm with continued high dose pressors. The patient went into v-fib arrest again, and at this point, both the cardiopulmonary surgeon and intensivist agreed the patient was terminally unstable. The patient was placed back on cardiopulmonary bypass and the family was consulted. Care was withdrawn and the patient expired. Discussion: This was a difficult case due to many confounding factors. I was the resident reading the initial EKG that was obtained in the ED but was not actively looking at the patient. The EKG looked concerning with ST depression in II, III and aVF and ST elevation in V1-2 with possible ST elevation in V3-4. The EKG shows a pattern that could represent an anterior wall MI but is not definitive and the computer interpretation was non-conclusive. When the EKG was presented to the attending ED physician, there was a brief discussion as to whether or not a STEMI alert should be activated. However, based on the patient’s presentation, history

page 11


The Fast Track and his insistence that he was having another MI, the alert was activated. Fall 2014

The case continued to be difficult due to patient agitation and the need to completely sedate and intubate the patient while in the cath lab. The intubation was done by an ED resident and anytime an unstable patient is cared for by ED staff outside of the ED, there is the added challenge of working in a foreign environment.

CASE 2: (2158) A 57yo male presents as a walk-in patient to the ED complaining of chest pain the radiates across the top of his chest and up into his neck. He reports his symptoms started about 15min prior to arrival to the ED and have persisted, prompting him to seek treatment. On arrival, the patient appears pale, diaphoretic and in some distress. He is difficult to elicit a clear hx from due to having some expressive aphasia, which is son notes is baseline for him and the result of an ischemic stroke (his son only knew it was due to a hole in his father’s heart). The patient did not take any medications prior to arrival in the ED. PMH: HTN, ischemic stroke (2008), seizure disorder (due to CVA) PSHX: none Social Hx: occasional smoker, denies EtOH and drug use, Exam: Vitals- BP 130/70, P 79, R 18, spO2 98%, pain 5/10 Gen- AOx3, anxious, in distress Pulm: CTA, no acute distress Cardiac: irregular, equal pulses ABD: soft, non-tender, non-distended Initial ED EKG: (2204)

page 12


The Fast Track

After resident evaluation of the patient, EKG (no previous) and consultation with the attending physician, a STEMI alert was called at 2204. - Initial testing: (not resulted prior to STEMI alert) CBC: 12/15.2/43/292 CMP: 139/3.6/102/19/21/1.0/149 Trop T: <0.010 CPK: 39 (low, no CKMB differentiated)

Fall 2014

ED Course:

ED/Hospital Course: Patient was dosed with ASA 325mg PO upon recognition of the STEMI. Heparin 5000u given IVP per cardio. Based on the patients symptoms, stable BP and IV access via 2 large bore IV’s, a single sublingual NTG (0.4mg) was given. Prior to administration, it was recognized that the area of concern on the EKG was consistent with an inferior wall MI. 2211: Patient taken to the cath lab 2230: Cath lab team arrives. During cath procedure, 100% occlusion was noted to the proximal RCA and a drug eluting stent was placed, reducing the occlusion to 0%. The patient tolerated the procedure well and admitted to the ICU for further observation. Hospital Labs: Trop T <0.010 -> 2.84 -> 4.96 -> 4.51 CKMB NA -> 133.0 -> 163.6 -> 105.1 Patient discharged home 2 days later on Brilinta, Lipitor, Lopressor, Lisinopril and ASA. And will follow-up outpatient to assess need for further cardiac intervention. Discussion: This case represents a classic STEMI in almost all aspects. The presenting symptoms and definitive EKG are typical on an inferior wall MI. The concern with this patient is whether there occlusion is in the RCA or not. If the lesion is in the RCA, medications that decrease cardiac preload such and nitroglycerin can potentially cause the patient to become hypotensive. In this patient, the lesion was in the RCA and nitroglycerin was administered, but in a controlled setting with a single dose in an otherwise stable patient who had good IV access. It is also interesting to note, the initial cardiac markers obtained on the patient in the ED were negative due to the rapid presentation of symptoms and subsequent initiation of care. Always be mindful that there is a delay in the elevation or cardiac markers from the initiation of cardiac injury.

page 13


The Fast Track

Fall 2014

CASE 3: An EKG is received from EMS and read at 1546. Faxed EKG (1546):

EMS then called by telephone reporting they were in bound with a 58yo female with a pmh of CAD with no prior stent placement, that was having chest pain with radiation to her left shoulder with associated nausea. She received SL NTGx3 with some relief of her pain. Their ETA was 20min. 1550: A STEMI alert is called after review of the EKG and EMS’ report. The cath lab was prepared for this patient with the plan that she would proceed directly to the cath lab on arrival to the hospital unless a rapid assessment in the ED necessitated immediate intervention. 1620: The patient arrived at the ED and EMS crew was meet at the ambulance entrance by the resident and attending. After a rapid assessment in the entryway, it was determined the patient was stable aside from her immediate cardiac concerns and she was taken directly to the cath lab on the EMS stretcher. No further ED evaluation, workup or treatment was performed. Hospital Course: The patient underwent emergent catheterization and was not found to have coronary artery occlusion constant with an acute MI. She was found to have mild to moderate vessel disease but no occlusion and was diagnosed with Takotsubo’s cardiomyopathy. After further discussion with the patient, she was under a significant amount of stress due to a legal situation and her chest pain occurred while she was at the courthouse. She had a pmh of HTN, HLD, CAD, smoking and depression/anxiety. Hospital Labs: CKMB: 10.2 -> 8.01 -> 5.57 Trop T: 0.236 -> 0.130 -> 0.068 page 14

The patient was discharged from the hospital two days later. Her chest pain had resolved and no re-occurred.


She had a cardiac echo that showed an EF of 50%. Her medication regimen was adjusted and follow-up care was arranged. Discussion:

Fall 2014

The Fast Track

The diagnosis of Takatsubo (Broken Heart syndrome) cardiomyopathy is a rare diagnosis that will initially present nearly identical to an acute MI but the symptoms and pathology are not actually caused by obstructive coronary artery disease. In this patient, her story and EKG were concerning enough to activate the cath lab. Her EKG though, was not typical of an acute MI, demonstrating near global ST segment elevation that was most noticeable in I, II, aVF and V2-6, with no reciprocal depression. Still, given the complete presentation by EMS, the activation of a STEMI alert was reasonable. This patient does however fit the classic presentation of Takatsubo’s, which typically occurs in a postmenopausal female during or follow significant stress (emotional or physical). While the exact disease process remains unknown, the result is an apical dilation of the left ventricle with decreased wall function. Prognosis of this disease is good, with near full recovery expected within months.

Resident Chapter Highlights SATURDAY 10/11 9am - Jeopardy (by Rosh Review)* 11am - Dental Lab 1pm - Membership Meeting and Elections* SUNDAY 10/12 10am - Residency Fair* MONDAY 10/13 10am - Airway Shoot Out CHIEFS COLLEGE - Monday 10/13 * denotes required attendance for travel reimbursement

page 15


The Fast Track

Fall 2014

Medical Apps for Medical Students and Residents too!

Nicholas Pettit, Ph.D., OMS-III @ OU-HCOM

T

housands of students will be making the colossal transition from the classroom to the clinic, and with that transition comes new responsibilities. Traditionally, students would be inundated with new information, statistics, facts and formulations all of which are fair game for those infamous “pimping sessions” which serves as a right of passage from the classroom to the clinic. While technology cannot necessarily rescue students from pimping, technology, such as apps on one’s smartphone, can play a vital role in a student’s education. Combining the ratings from the iTunes app store as well as data provided from a Harvard Medical School survey, below are 10 applications that are must haves for students hitting the wards.1-3 The best part of this list is that one can go to their local grocery store, buy a $10 gift card to iTunes and afford all of these valuable apps (in addition to earning gas points). Keep in mind, little evidence exists about using applications in regards to patient safety, and it is recommended that one always confirms their finding with other non-app sources such as your attending or senior resident.4 1. EyeMd – Need a flashlight to check to pupillary response? Check! Need an eye chart or illustrations helping determine pupil size? Check! This inexpensive application has an easy-to-use interface that is a must have application for visual acuity exams. Price: $0.99 on iTunes. 2. eBlood Gas/MedCalc/CliniCalc – Writing an article for a scientific journal, or just want to impress your attending by calculating some obscure number like the APACHE score, or need help figuring out acidosis/alkalosis calculations? Check out one of these medical calculator apps. Prices: eBlood Gas: $1.99 on iTunes; MedCalc: $1.99 on iTunes; CliniCalc: Free on iTunes. 3. Medical abbreviations – No hablo español! That is what Google translator produced when I asked for a translation of “I don’t speak Spanish.” While this app doesn’t translate languages (those apps do exist however), this app helps translate the foreign medical

page 16


jargon that I have been experiencing on the wards! The best part, its free! Price: Free on iTunes.

Fall 2014

The Fast Track

4. UpToDate – The good? Probably the most famous piece of software to-date is now useable on both iPhones and Androids. The bad? You need a subscription, usually offered through your hospital or their library system. Price: Free to download but the subscription price is variable. 5. Micromedex - Epocrates is probably the more famous drug-reference app for handheld devices. However, Micromedex will give any similar app a run for its money with it’s simple-to-use interface. Price: Free on iTunes. 6. Medscape – This app is among the most downloaded medical apps to date. While this app has some features that parallels Micromedex, it also has some information about those infamous zebras, information about procedures, and provides CME for physicians. Price: Free on iTunes. 7. NEJM This Week – Bored on a rotation? Read all the required readings and have nothing but your iPhone and Angry Birds? Take a look at this free app to help both pass the time and enhance your knowledge base. Price: Free on iTunes. 8. iRadiology – Hey random OMS-III, describe this fracture on this radiograph! (In a stuttering voice, “I…….uhhh…..”). This app allows one to study over 500 unique cases, even without an Internet connection. Price: Free on iTunes. 9. Relax Melodies – Spending your first night of call sleeping in a 6’x10’ room adjacent to the residents’ lounge and unable to sleep? Are those pesky interns that should be on the floor spending an hour playing Xbox with the volume at the maximum level? While this app isn’t a medical app per say, many applications designed for the general public may have great utility for us medical students. This app has 33 relaxing tones, such as “Thunderstorm” or “Slow Waves” which can help drown out undesired noises. Price: Free on iTunes. 10. Lose it –This app helped me lose 25 pounds during my PhD work as I neglected my diet. This app allows one to track calories, exercise and weight, as well as set long-term goals. Those 3 free slices of pizza scarfed down at the free lunch lecture actually cost almost one thousand calories! page 17


The Fast Track

Fall 2014

Hyperlipasemia associated with acute, colonic diverticulitis Dhimitri Nikolla OMS IV @ LECOM-Erie Brandon Kramer DO @ Saint Vincent Hospital ABSTRACT: INTRODUCTION: Abdominal pain is a common complaint in the emergency department and we often use lipase levels to rule out pancreatic causes of abdominal pain. However, hyperlipasemia is not entirely specific to pancreatitis and has been seen in other conditions such as diabetic ketoacidosis, ruptured ovarian cyst, toxic epidermal necrolysis, Stevens Johnson Syndrome, aripiprazole use and Gullo’s Syndrome or benign hyperlipsemia. CASE REPORT: The patient is an 86-year-old female who presents to the emergency department with stabbing suprapubic abdominal pain and hyperlipasemia. Computed tomography (CT) of her abdomen showed acute colonic diverticulitis without any evidence of acute pancreatitis. CONCLUSIONS: Here we present a unique case of hyperlipasemia associated with acute colonic diverticulitis without any radiographic evidence for concomitant pancreatitis. Although hyperlipasemia is sensitive for pancreatitis, clinicians should be aware that other non-pancreatic causes exist. INTRODUCTION: Colonic diverticulitis is inflammation of colonic diverticula, which are outpouchings of mucosa through the muscular wall of the colon. About 50-70% of people above the age of 80 have diverticulosis and 80% of those with diverticulitis are greater than or equal to 50 years of age. Clinically, it presents with obstipation and abdominal pain, usually in the left lower quadrant. CT is the test of choice with a sensitivity of about 93 to 97% and specificity of 100%.1 Pancreatitis is an inflammatory condition of the pancreas accounting for greater than 220,000 US hospital admissions per year.2 It presents with abdominal pain and nausea associated with hyperamylasemia and hyperlipasemia, usually three times the upper limit of normal.2 CT can be used to confirm the diagnosis (sensitivity 90%, specificity 100%), to stage severity of the pancreatitis and to rule out other abdominal pathology.2,3 Here we present a unique case of hyperlipasemia associated with acute colonic diverticulitis without any radiographic evidence for concomitant pancreatitis. CASE REPORT: The patient is an 86-year-old female who presents to the emergency department with suprapubic abdominal pain that began three days ago. The pain began as “soreness,” but has gradually and progressively become acutely painful and associated with nausea. It is stabbing in quality, persistent and radiates from the center out to her right and left lower quadrants. Moving makes it worse, while sitting still makes it better. Her pain peaked at a 10/10 about three hours prior to arrival at the ED. This exacerbation of her pain is what prompted her to come in and be evaluated. Today she had one episode of non-bilious vomiting. She also admits to constipation for the last three days and one episode of diarrhea just prior to examination.

page 18

On review of systems, she denies hematochezia, melena, fever, chills, back pain, discharge, vaginal or urethral discharge, increased pain with food, incontinence or urinary retention. Her past medical history is significant for hypertension, hyperlipidemia and diverticulitis. Her current medications include aspirin, amlodipine, potassium chloride, atorvastatin, metoprolol, calcium, fish oil and a multivitamin. Past surgical history includes a hysterectomy and appendectomy. She denies any known allergies, any


The Fast Track On physical exam, her vital signs are 36.7°C, pulse 75 beats/min, respirations 18/min and blood pressure 202 / 82 mmHg. Her heart exam reveals a regular rate rhythm, but a 3/6 systolic murmur over each listening post. Her lung exam reveals diffuse faint crackles. Her abdominal exam reveals normoactive bowel sounds, moderate suprapubic tenderness, soft to palpation, no rebound, no Murphy’s sign and no CVA tenderness. She has strong distal pulses and no edema in her legs.

Fall 2014

family history and any hospitalizations in recent years.

Her lab results are remarkable for WBC 12.9 x 103 mm3 and Lipase 278 U/L. The metabolic panel, liver function tests and lactic acid are all within normal limits. A chest x ray shows no free air under the diaphragm and a CT Abdomen reveals acute colonic diverticulitis without any evidence of pancreatitis (Figure below). The patient is diagnosed with a recurrence of her diverticulitis and intravenous antibiotics, ciprofloxacin and metronidazole, are begun. She is given Zofran for her nausea, morphine for pain and intravenous fluids for hydration. She is admitted to the hospital to continue intravenous antibiotics and pain control. Figure > A slice the patient’s abdominal CT scan exhibits diverticulitis. The rest of the scan is negative for pancreatitis.

DISCUSSION: Although an elevated lipase is thought of as pathognomonic for pancreatitis, there are several documented, non-pancreatic causes of hyperlipasemia. A literature search for hyperlipasemia brings up case reports with the finding in patients who had diabetic ketoacidosis (DKA),4 ruptured ovarian cyst,5 pediatric patients with toxic epidermal necrolysis,6 Stevens Johnson Syndrome,6 patients taking aripiprazole,7 and Gullo’s Syndrome or benign hyperlipasemia.8 Several mechanisms have been proposed for hyperlipasemia associated with these conditions; however, a definitive cause has not been elucidated. Metabolic conditions, such as DKA, can cause direct injury to the pancreas causing it to leak enzymes.9 Impairment in renal clearance may contribute to the elevation in pancreatic enzyme levels.4,9 Similarly, portal congestion and poor hepatic function have been suggested as mechanisms for reduced clearance.9 Lastly, ectopic production from non-pancreatic tissue may contribute to elevated pancreatic enzyme levels, such as from lung cancer, ovarian cancer and normal ovarian tissue.5,9 Although the sensitivity of CT for acute pancreatitis is only 90%, it is very unlikely that our patient would have coinciding pancreatitis and diverticulitis, especially without epigastric pain and no history of alcohol abuse or any other obvious causes. CONCLUSIONS: Hyperlipasemia has been previously reported with non-pancreatic conditions. We present a case of acute colonic diverticulitis associated with hyperlipasemia and no radiographic evidence of pancreatitis. Although these cases are rare, clinicians should know that hyperlipasemia is not entirely pathognomonic for pancreatitis. page 19


The Fast Track

Fall 2014

Perspective from Above: HealthNet and Aeromedicine in Today’s ED By Sasha Rihter, OMS-III @ WVSOM hey don’t look so big in the sky. Loud but nimble, helicopters seem to grow in girth as they begin to land. But seeing the oversized, heavy blades sag to a stop and the capabilities of these machines and their crews give a new perspective on these previously untouchable dots in the sky. Seeing them in person, you intuitively imagine the feeling of hurdling through the sky towards trauma scenes at speeds of 150 mph, climbing 8.1 m/s into the air in a flurry of wind and noise towards the sirens and smoke. From incredible feats of bravery and resourcefulness in snow storms and mountains, to turning a several hours long journey to a matter of minutes, helicopter crews redefine what it means to work in modern emergency medicine.

the service decreases the overall number of calls for help. Yet, it is exactly these remote areas where ground transport can take longer than air that aeromedicine becomes the most vital. Collectively, these issues make it difficult to calculate the overhead cost and efficiency of maintaining such services. Flight operations, like HealthNet in West Virginia, work to expand the range of hospital services by providing a fast and efficient means of patient and medical crew transports over long distances or impassable areas of land. HealthNet began in 1986 as the brainchild of West Virginia University Hospitals (WVUH) in Morgantown and Charleston Area Medical Center (CAMC) in Charleston, in response to the dwindling funding of the then current helicopter services in the mountain state. Today, the organization has expanded to eight separate stations serving the states of West Virginia and portions of Kentucky, Ohio, Virginia, Maryland, Pennsylvania, North Carolina and Tennessee. The need for such services is not limited to the mountain states. By expanding the reach of regional medical centers, helicopter transport becomes crucial wherever there are barriers preventing the sick from accessing care in their state.

The industry of aeromedics began in 1928 with the Flying Doctors of the Australian outback and has since become a seamless blending of modern technology with responsive medical action. These teams of professionals help provide emergency medical care to the nearly 25% of the US population living in rural America and doing their part of the national effort for improvements in medical access to the growing underserved. This impressive expansion of hospital services comes with a high price tag, costing insurance companies or the patient thousands of dollars. In addition, Whether in remote regions of the globe or the sparse distribution of homes covered by vast expanses of service areas, companies page 20


Fall 2014

The Fast Track

and hospitals coordinate efforts to serve as many as possible. With the growing use of helicopters for transport in emergency medicine, it becomes increasingly important for physicians to familiarize themselves with the benefits and proper utilization of these resources. Participating in rotations and training in organizations like HealthNet helps prepare students and residents for future careers in emergency medicine, OB/ Gyn, and primary care, especially in remote or underserved areas. For those interested in understanding the unique perspective these services provide, Mike Peterson DO, HealthNet’s medical director, recommends a “team-orientated attitude” as a must. While the opportunity to rotate, shadow, and train with their program is “open to anyone with a willingness and desire,” interested students and residents need to have an “eagerness to learn, be thick-skinned and understand failures will happen and how to overcome them.” For the lucky few chosen to train through the handson experiences, young physicians will learn when it is appropriate to use flight operations, preparation for transport and arrival, as well as training in field and situational triage. “Truly,” Dr. Peterson warned, “applicants need to be willing to learn and do anything to help patients and put 100% of themselves and their effort into understanding and cooperating in the team-dependent environment.” For Ryan Quinn, OMS IV, a former participant, the adventure of being “3000 feet in the air… giving state of the art emergency medical

care to a transplant rejection patient all while moving 150mph in the dead of night,” gave him a “new perspective on how diverse rural care can be.” Of his rotation, Ryan says that “providing access to critically ill patients who don’t have time to spare was equally rewarding and exhilarating when being done with the staff of first rate nurses and medics on the HealthNet team.” Beyond the initial child-like fascination, the interest in aeromedicine goes deep to the heart of why many of us love emergency medical work. As a blend of human ingenuity, teamwork, and efficiency, helicopter transport crews move fluidly through moments of despair and anguish while pushing the boundaries of medical science’s capabilities. These teams of men and women have a sense of camaraderie and interdependence embedded within all aspects of their operation. The skills learned in this unique environment help build the characteristics crucial to any setting: a balance of teamwork, problem-solving and a strong commitment to excellence. There is little room for error and safety of crew and patient is at the forefront at all times. Being skilled, caring, and efficient under such intense circumstances prepares young physicians for the realities of critical care and gives them a new perspective on aeromedicine as the next step forward in emergency medicine. For more information about HealthNet check out healthnetaeromedical.com. + page 21


The Fast Track

Dealing with Death as a Medical Student

Fall 2014

By James Chapman, MSIV, AZCOM

U

nfortunately death is a reality in emergency medicine. We chose this career with the goal of helping others and saving lives. When someone we are taking care of dies it can be a hard situation. How should we deal with this? There is no class or lecture that can prepare us for

these circumstances. Should we express grief or should we remain emotionless? Although there is nothing that can prepare us for this difficult challenge, we will all experience it at some point in our careers. An experience I recently had was eye opening and I want to share it so that others may benefit from it. I remember the first time one of my patients died. During my third year of medical school, the patient, who I will call Mr. Jones, had a heart attack and I was assisting him in the catheterization lab. During the procedure Mr. Jones was experiencing more pain and discomfort. He became agitated and was not being cooperative with the physician. His condition deteriorated and it was decided that he would need to go for emergent bypass surgery. He was intubated on the table and transported to the ICU. Within five minutes of being moved to the ICU a code was called and Mr. Jones died. What did I learn? During the procedure, the physician became aggravated when Mr. the patient was uncooperative. In a manner of unprofessionalism he used expletives and words to demean the patient even though he was in the middle of a heart catheterization. After this exchange, a nurse approached the other side of the table and soothed the patient with kind words and a gentle tone. At the time I didn’t realize it, but these were the last words the patient would hear. The nurse’s example taught me to treat others with respect no matter the circumstance because one’s comments may be the last kind words spoken to a patient. Following the death of Mr. Jones we went to discuss what had happened with his family. There are many different strategies that physicians use to inform the family. Take your time with them and be a willing listener. Be direct but sensitive when answering any questions. Jonathan Marrow, an emergency medicine consultant in the United Kingdom, gave some excellent advice in an article titled “Telling Relatives that a Family Member has Died Suddenly.” In this article he dives into many aspects of how to share bad news and I would highly recommend reading it. Mr. Marrow gives the following advice in regards to talking to family members:

page 22


1) Ensure privacy in a nonclinical and comfortable environment

2) Make time, do not be rushed or disturbed

3) Make sure the bereaved person is not left alone after the interview

4) Check your facts (Identification of deceased and identity the relative, be ready to

answer likely questions about the patient)

5) Check your appearance (take off blood stained gloves, gowns, etc.)

Fall 2014

The Fast Track

I have also found it helpful to check in on the family once they have had some time to grieve. As medical students and residents it is also beneficial to take time to discuss what occurred with the attending and other members of the team. This is the proper time to discuss what could have been done differently and express any concerns one may have. The most difficult part of dealing with death for me was coping with my own personal grief. You will always wonder if there was something more you could have done. It is an important question to ask in order to become a better physician. Following Mr. Jones’ passing, I attended to other patients but his death weighed heavily on my mind. As I looked at the stoic expressions of the many experienced physicians, I wondered if I was the only one feeling grief. Through many more experiences, I have found that each individual deals with grief in their own way. Some physicians I know take some time to cry and pray, while others keep busy to keep it off their minds. Our responsibility as students and residents is to figure out how we can properly deal with grief and continue our daily tasks. We must be able to provide the best care possible to the patients we are still responsible for. Dealing with death may or may not become easier the more we experience it. Regardless of how we deal with death we must respect our patients, both those that have passed and those that are still here. The family deserves to be formally addressed and the events surrounding their death should be explained. We must find the time to personally grieve. We need to be completely focused on our patients. Spending a few moments to manage our grief will benefit all those around us. The most important advice I can give, especially following a death, is to provide one hundred percent effort to each patient and their family. As you heed this advice, you will find compassion and be successful throughout your medical career.

page 23


The Fast Track

Fall 2014

Ghana: An Experience Through sidHARTe By Frederick Davis, DO

One

of the most rewarding experiences of International Emergency Medicine is the variety of people you can help with the resources you bring with you, for the time you are there. However, one of the most frustrating aspects is that once you leave, so do many of those resources. One program has been developing a different approach to missions abroad by working on systems development. This approach has focused on ways to design a health care delivery system that can continue treating patients once the physician leaves. This group is Systems Improvement at District Hospitals and Regional Training of Emergency Care, or simply sidHARTe. sidHARTe is a joint initiative between Columbia University and the Ministries of Health in Ghana and Rwanda. The long-term goal of this initiative is to improve the emergency medical care in resourcelimited settings in sub-Saharan Africa, starting at the district level. The program is built on the belief that access to emergency care should be universally available and accessible to all. Toward this end, sidHARTe has created an emergency care curriculum directed at the diagnosis and treatment of patients in the rural hospital setting. The curriculum was based on the model of the World Health Organization (WHO) and other international resources and revolves around five core modules: airway/breathing, circulation, mental status, severe pain, and dangerous fever. The Mampong District Hospital houses 154 beds for the 120,000 people in the area it serves. There is a separate male and female ward with the children’s ward being shared with the female ward. There are two OR suites, mostly kept empty because there are no surgeons available. Nurses here play a primary role in evaluation and treatment of patients upon arrival. What is also unique to this system of care is their focus on preventive care and treatment. Throughout the hospital there are posters of public messages on brushing your teeth, the effects of diabetes, signs of malaria and tuberculosis, and promoting of safe sex and abstinence. The hospital also has an area that they use as an infectious disease clinic where they could have HIV counseling and antiviral therapy could be given. HIV detection and treatment has been one of the country’s initiatives and they have put forth that anyone diagnosed with HIV would be given medication for treatment at no cost.

Emergency Medicine... is still a new concept in many places

page 24

The focus of my time there was spent helping run their Casualty Ward. Emergency Medicine is the realm of medicine that is still a new concept and many places still do not have any developed emergency treatment or evaluation areas. Instead,there is an area of the hospital that treats walk-ins and start treatment as needed. At Mampong District Hospital, this area was the Casualty Ward. This area iss a wing of the hospital that has a small area with an automatic BP cuff for triage, a room with four female beds and three cribs for children, another room with three male beds, the ever popular hall beds that are found as the place got busy, and one room that had two beds for resuscitation. The resuscitation


The Fast Track

The cases we would see daily would range from typical medical complaints of weakness and diarrhea, to various injuries from trauma. Daily there would be a patient suffering from some degree of malaria or typhoid. Other cases I saw included asthma, pneumonia, hypertension, diabetes, CVAs, trauma, as well as snake bites and ingestions. In the four weeks I was there, I saw patients ranging from a one month old with pneumonia to a woman over 100 years old with a UTI. We saw overdoses on insecticides like DDT and multiple cases of malaria even to the extent where one child who presented with lethargy was found to have a hemoglobin of 3.2 and hemocrit of 8.0. We treated cases of trauma attaching a thumb back on an eight year old who chopped it off with a machete while gardening, to a two year old with over 30% 2nd degree burns after boiling water fell on him, as well as a mass casualty incident when a bus overturned and we got 13 patients at once from the accident.

Fall 2014

room was also where many of the daily procedures were conducted, including wound care and suturing.

When you are practicing in another medical system, you are limited by what tools you have available. At the same time you are limited by what tests you have available. We had access to testing blood sugar and urine analysis as well as blood tests for malaria, but at the same time the only other testing that was accessible was a CBC. No electrolyte panel was available for those that appeared dehydrated or weak after multiple bouts of diarrhea or ABGs for those with elevated glucose levels or continued SOB after treatment. This limitation requires you to rely more on your physical diagnostic skills, which we sometimes neglect practicing in a developed country where one can easily get a CT scan to see what is going on inside. The majority of my time there was spent on educating the nurses on how to evaluate patients while giving lectures on specific topics of commonly occurring presentations in that area. As can be expected, my time there was extremely rewarding. Not only did I gain insight into the way medicine was practiced in the hospital setting in Ghana with limited supplies and access to different medications, but I also got to see the staff adapt and learn with each new patient. From teaching bedside rounds and seeing how they would be more astute in their evaluation and management, to how they taught me the various treatments for diseases like malaria and typhoid that are less common here in the US, to how we all worked together as a team to provide the best care possible, what I learned was that we were limited in our care abilities. The development of sustainable methods through education and sharing of resources, such as through sidHARTe, needs to be the next step in International Medicine. Without a continued renewable resource that education provides to train the following generation, the efforts spent by medical professionals in developing countries will not be sustainable in the long term.

page 25


The Fast Track

Fall 2014

Brain Doping An opinion editorial on the use of Adderall by Medical Students Giles Gifford, OMS-III @ Pacific Northwest University

Ipractice t is common for medical schools

page 26

to mandate a code of conduct or an honor code among attendees. Yet as in many aspects of life, this vow has become open to interpretation and grey areas can be found, such as with the use of performance-enhancing drugs. While there are no formal studies that report the incidence and consequences of the use and/or misuse of amphetamines specifically by medical school students, the illegal use of these drugs in undergraduate students ranges from 10 to 30%.1,2,3 In today’s extremely competitive medical school environment, some students look for any possible advantage they can muster to better position themselves for success and the future they desire. This gives rise to the question: Is it a violation of the Honor Code for medical students - future physicians - to use prescription drugs for which they have no diagnosed indication as an aid to help learn the overwhelming amount of information that is required? Should all medical students be kept to a strictly-enforced standard of being able to rely only on their natural abilities, rather than those that are pharmacologically enhanced? Should we “drug test” medical students for the use of amphetamines? On admission, before and after Board Exams, before and after every exam block, or every

day? After beating testicular cancer that had metastasized to his lungs, lymph nodes, and brain, Lance Armstrong went on to win the Tour de France an unprecedented seven times, setting the world record and becoming an international hero. Just a few years later, after his admission of blood doping, he was stripped of all seven titles and received a lifetime ban from competitive cycling. Those who saw or heard about Oprah Winfrey’s interview with Armstrong had to formulate their own opinions of just how important his admission of blood doping was, and what the underlying implications were for him, for sport, and for society. Some believed that it was “no big deal” while others were outraged. At this point, we were all forced to make a decision: did you think that Lance Armstrong was a cheater or was he cheated? Did he feel that he needed to break the rules just to remain competitive? I ask us all to consider that a parallel may exist in medical education. Adderall®(generically: a m p h e t a m i n e + dextroamphetamine) use is banned by the U.S. AntiDoping Agency which regulates Olympic sports as well as Major League Baseball, basketball, soccer, the NCAA and the NFL, all of which have classified it as a “performanceenhancing drug.” Should

medical students be held to the equivalent or a higher standard than competitive athletes? Should we require screening for this or all types of “performance-enhancing drugs” routinely? Adderall® is a sympathomimetic. While the exact mechanism of action is unknown, it stimulates CNS activity by increasing the release, as well as blocking the reuptake of both norepinephrine and dopamine. It has Black Box Warnings that caution about its high potential for abuse and dependence, as well as potential for non-therapeutic use or distribution to others. While there have not been any formal studies that have shown that the use of Adderall® has improved medical students’ class rank or Board scores, the drug can be used as a study aid, allowing students to achieve levels of concentration and focus that they perceive are needed to study effectively for prolonged periods. This can be very enticing to a medical student with a heavy course burden and frequent exams. Some institutions have policies requiring dismissal after just one failure, making the stakes even higher. As the number of medical students increases annually and the number of residencies available remains static, many students look for any advantage to score well on


exams. It is not very difficult for a student to access these drugs. The risk of detection is relatively low, as drug tests will be negative after about four days of abstinence.4 For some students the consequences of testing positive are too great, others are willing to accept these risks, as they perceive the risk of failing a test or a course to be far greater. To solve this dilemma, some students decide to visit a doctor to obtain a prescription for the drug using the indication of adult ADHD or narcolepsy. Medical students are in a unique position in that they are trained to know what questions are asked to make the diagnosis of ADHD and more importantly what answers to give if they wish to acquire that diagnosis. If a student hasn’t had this training but still wants to access this drug, finding a detailed tutorial on how to obtain a prescription is as simple as doing a Google search.5 Just recently this issue has gained some national attention. On August 28th, 2014 the Coalition to prevent ADHD Medication Misuse (www.cpamm.org) was launched and on September 02, 2014 the American Academy of Family Physicians joined its ranks.6 Time will tell if other professional organizations or medical institutions will also support this coalition. According to the National Institute of Health, approximately 4% of Americans are treated for adult-onset ADHD every year, and there are students that legitimately suffer from this and other conditions that require the use of this medication. So why does this matter? Why should we care that some of our colleagues are using amphetamines while in medical school? Similarly to

how marijuana is commonly referred to as a “gateway drug,” this argument can also be made for amphetamines. After the completion of residency, many physicians will obtain a license to prescribe Schedule II medications. As mentioned, two of the Black Box Warnings for Adderall® are dependence and abuse. A physician who depends on Adderall® to get through his or her shift will likely develop tolerance to its effects and eventually require higher doses, perhaps even turning to different stimulants to continue to function at this level. What’s more, these doctors will most likely continue to prescribe this medication to others who have also identified this perceived need to help them get through medical school. As with the Lance Armstrong controversy, this subject matter is sensitive. Some view the situation as “black or white,” while others prefer to look at it in shades of grey. Consider the student who obtains Adderall® without a prescription but takes it only during a test week to help remain focused - this behavior can at least be understood if not unofficially condoned. It’s the “ride as fast as the other riders” argument, or evolutionarily, those who are fittest will survive and whomever has the advantage will win the race. Doing this might bring a clear advantage to

those who are willing to risk the low probability that they will get caught (no school publicly acknowledges routine testing for amphetamines on or around exam days). Perhaps a more controversial question: What i s the medical community’s stance on students who fabricate or exaggerate symptoms in order to obtain a prescription for Adderall®, which is then used only as a short-term study aid? Thus far a blind eye has been turned, but this may cause more long-term harm than short-term benefitIt may be important to at least recognize that this is an issue that students are facing in their daily studies and while preparing for school and Board examinations. It sometimes seems that the only voices to be heard on the matter are from students referring one another to the “local Adderall® Doctor.”

Fall 2014

The Fast Track

Acknowledgements: Special thanks to William Elliott MD, PhD for his continued support and guidance.

page 27


Pimpology 101 The Fast Track

Fall 2014

Top Things to Know on your EM Rotations Danielle Turrin, DO, MS @ Good Samaritan Hospital

Pericarditis

Inflammation of the pericardium producing symptoms including chest pain, tachycardia, dyspnea. Chest pain is characteristically described as substernal, pleuritic, worse with lying flat, relieved by leaning forward. Patient will often have a low grade fever. A pericardial friction rub is pathognomonic. May also develop pericardial effusion, some progress to cardiac tamponade Causes: Infectious- viral, bacterial, fungal, TB Immunologic- RA, SLE, scleroderma, rheumatic fever Uremia Post-MI (Dressler’s Syndrome) Trauma Paraneoplastic Drug-induced (i.e. INH, cyclosporine) Post-radiation therapy Post-cardiac surgery (postpericardiotomy syndrome) EKG Stages of Pericarditis: Stage 1-Diffuse concave upward ST elevation and PR depression with reciprocal changes in aVR (above) Stage 2-Normalization of ST segment; generalized T wave flattening Stage 3-Flattened T waves become inverted Stage 4-EKG returns to normal Management: Treatment for specific causes of pericarditis is directed according to the underlying cause. NSAIDs are the mainstay of therapy, treatment should last 7-14 days. Surgical procedures may be necessary for patients with large effusions and include pericardiectomy, pericardiocentesis, pericardial window placement, and pericardiotomy.

page 28

For more quick, easy, factual pimp-question information don’t forget to check our Pimpology 101 in our next issue of the Fast Track...


The Fast Track

SATURDAY 10/11 Noon - Skills Lab 2pm - Rapid Fire Lectures

Fall 2014

Student Chapter Highlights

SUNDAY 10/12 10am - Residency Fair 2pm - Student General Membership Meeting and Elections MONDAY 10/13 8am - Airway Shoot Out 2pm - Mock Interviews

Looking to get more involved in Emergency Medicine? Run for a position with your ACOEP Resident or Student Chapter! RC Elections are Saturday, October 11th at 13:15 SC Elections are Sunday, October 12th at 14:00

READY TO SOCIALIZE IN VEGAS? WE ARE! Check out our evening events: Saturday, October 11th @ 6p - Margaritaville sponsored by TeamHealth Sunday, October 12th @ 7p - ACOEP Welcome Reception Sunday, October 12th @ 9p - Night Out with EMP Monday, October 13th @ 7p - FOEM Gala

For more information and locaitons, checkout the schedule of events at the RC and SC tables!

page 29


The Fast Track

Fall 2014

I’ll never forget

my first day of recess as a first-grader. When the teacher told us to go outside I ran straight to the basketball court, but to my amazement no one else was there. “Maybe I’m just faster than everyone else,” I thought, but I looked around and all the students were on the other side of the playground. I ran over and saw everyone lined up to play a game with four people all in separate squares drawn on the ground, bouncing a ball. I had never seen kids play this strange game. Of course I got in line—I wanted to fit in—and before long, it was my turn. Like a lion targeting a baby gazelle with a broken leg, the girl in the first square quickly pounced and threw the ball at my feet. Naturally I tried to kick it. The girl started to laugh and shouted, “You don’t kick the ball in Four Square. Ha ha ha . . .” Then everyone else joined in. I remember feeling lost and as though everyone in the world knew what was going on except for me. After years of therapy I was able to forget that traumatizing recess.

How to stand out... in a good way

Let’s fast-forward to my first day of medical school. I walked into class and looked around to see every other stuCameron Meyer, OMS-IV at WVSOM dent already in their seat and everyone talking about how they had a double PhD in biochemistry and theoretical astrophysics. I quickly found an open seat next to a sweetlooking girl. We greeted each other and she asked, “What are you going into?” Thinking to myself “am I already supposed to know this?” I responded “I don’t know, maybe the ER.” Then she hit me with a bunch of questions that appeared to be some foreign language. “Do you prefer a Miller or a Mac? What gauge of tubing should you use in a central line? What are the side effects of succinylcholine?...” I suddenly had a flashback of the little girl laughing at me, “You don’t kick the ball in Four Square. Ha ha ha . . .” If you’re anything like I was you don’t know what it takes to get from being a first year medical student to your dream job in emergency medicine. I definitely don’t have all the answers to get you to where you want to go, but I have been lucky enough to get to know some really smart people who have landed that dream job or residency program. I want to share with students some of the great advice I received from them. First, I know this sounds obvious, but you need to spend so me time in the emergency room. This is the best way to know if you want to spend the rest of your life there. Most schools allow you to do an emergency medicine rotation in your third year, but many schools limit the amount of time you can spend in any one specialty. If you are in this position, make sure you only do one ER rotation during your third year. That way you have more opportunities for audition rotations in your fourth year.

page 30

Some students have a say in their third year rotations and often ask, “When should I do my third-year ER rotation?” Before I answer this question you need to decide if you are 100 percent sure emergency medicine is what you want to do. This is what will determine when


you should try to schedule your rotation. If emergency medicine sounds good but you are still unsure, you should schedule your ER rotation early in the school year. That way if you decide that it’s not your thing you can quickly change your focus. If you realize in your last rotation that you hate the ER, it’s too late. Your audition rotations for fourth year will have already been scheduled and it might be too late to change them. However, if you know that the ER is your one true love, do that rotation last. Then it will be fresh in your mind and you will be at the top of your game for audition rotations.

Fall 2014

The Fast Track

What about the rest of your rotations? Understand that ER physicians see EVERYTHING. You do need to know how to handle trauma (of course), but you also need to know how to handle just about everything else. Where does a person go when they are sick and realize their physician’s office is closed? Where does a pregnant woman go when she slips in the tub and wants to check on her baby? Where does someone take their demented grandma? Make sure that you learn everything there is to know during your other rotations because emergency physicians need to be well-versed in just about every medical specialty to make sure that their patients get the treatment they need as quickly as possible. Another important principle I was taught is that one of the best qualities a medical student can possess is humility. Understand that you are less valuable than a nurse, a CNA, or any other staff member. The hospital can function without you, but it cannot function without its nurses or even housekeeping. Be nice to them and realize that most nurses have great relationships with their physicians and can have a say in your future. No matter how bright you shine or how great you get along with your attending physician, I promise that you can kiss that letter of recommendation or residency spot goodbye if you don’t treat every staff member with respect. If they see that you are humble they will be willing to help you look better in front of the doctors. Spend time with the nurses—even make them cookies from time to time. It will go a long way. One of the most difficult obstacles we face is standing out in a sea of prospective ER physicians. Good grades are important, but in reality it is nearly impossible to compare students’ grades from different schools. So aside from board scores how can you stand out enough to make sure you get an interview at your favorite program? Attend conferences. Show the leaders of these programs that you are serious about emergency medicine. Of all the ACOEPStudent Chapter members, less than five percent attend conferences; not many students are willing to sacrifice their time and money to attend. But, at conferences not only do you get to hear lectures that actually interest you, and go to labs that are practical and fun, but you get to spend quality time with the people that make the decisions at your future residency program. The program directors, the residents, and other important staff members are at conferences largely to look for prospective applicants. They all have a large say on whether or not you get into their program and there are few other opportunities for you to get to know them on a personal and professional level. Medical school is tough, but so is Four Square. Make sure you know how the game is played so you don’t make a complete fool of yourself. Ask others for advice; the majority of us started this path lost and confused, and most everyone ahead of you wants to help you succeed. The more you know now the better you will perform when it’s time to jump into the first square.

page 31


The Fast Track

Fall 2014

Making the Most of An Interview: By Andy Little, DO @ Doctors Hospital

S

ome of the most enjoyable moments in the process of choosing a residency is the day, the hour and the minute you get an email saying, “We would like to offer you an interview.”

Unfortunately this happy moment turns can turn into a ball of emotion as you begin to think of what can happen or go wrong. Interviewing is the last important piece in finding the right program and has the most weight of a program choosing you as a future resident. Underestimating the power of this experience will not be a good choice on your part, so read carefully. Like many of the articles I write, I’ve elicited the help of some friends from around the country, to give you a well-rounded approach to approaching your future emergency medicine interviews. Below is what we felt was important for you to remember. Choose your interview date quickly. From the moment you receive that email you are on the clock with that program. You should quickly look at your calendar and choose an available interview date. Programs know when they offered you and interview and if you take a while to choose a date that may be the beginning of a bad impression. After you have chosen a date, don’t be afraid to call to find out more about the interview process at a specific program. Will you be doing one on one interviews (and if so how many), or will it be a panel style and who do they usually have you interview with? (chief residents, program directors, core faculty, etc.) Will there be a dinner the night before you should plan on attending, or a lunch afterwards? Also, will you be attending didactics or other learning experiences during your interview? The best way to prepare for your interviews, is to practice. Whether it’s with a clinical preceptor, through your school or at a conference, take the time to practice interviewing. Most physicians remember their residency interviews and can be a great help. This way you are not going into your interview inexperienced.

page 32

Once the day arrives, start off right by being well dressed. I know in emergency medicine we usually wear glorified pajamas but the interview should be in business dress, unless specifically told otherwise. So get out the suit you wore when you interviewed for medical school, freshly pressed dress shirt, tie and shined shoes. It is amazing how many show up in khakis, polo’s or


The Fast Track

Bring a copy of your CV. Even though programs have your ERAS application, it still is a good idea to bring a copy of your CV. If you are doing panel interviews, it won’t be as important, because each person will probably ask you about different portions of your application. But, for one on one interviews it can be extremely helpful, as not all interviewers have looked at your ERAS information in detail and your CV is organized in a way that the information they are looking for is easy to find.

Fall 2014

others expecting to be taken seriously. Be smart and don’t make this mistake.

Be direct in your answers. You will be asked some hard questions and they will expect good answers. Be sure to be concise, to the point, but thoughtful. I know this sounds like a lot, which is why you should have practiced prior to interviewing. If there is some glaring issue on your application, it will be brought up. Be honest and open when you talk about it, this will be your opportunity to calm any concerns a program may have about you. Know that clinical knowledge is fair game during interviews. Whether it’s interpreting EKGs, x-rays, ABG’s or having to work through a case, there will probably be some form of basic clinical knowledge involved in your interview. Whether you get it right or not is not always important but how you handle it is. Be sure to have questions for the program. Even though you could probably learn most things about the program by going to the web or in the interview day itself, be sure to have a few good questions about the program. This shows some interest on your part, and allows for you to also get answers to questions they may have left out. Some examples are questions regarding specific rotations, specifics about working in the ED, possible electives, how didactics work, etc. When you’re done with the interview, be sure to follow up. Whether this be in the form of a email, phone call, or a thank you card this shows gratitude and your interest in the program. This can go a long way with program coordinators, program directors and is also polite. But, don’t go overboard and send flowers, buy a television or similar as this may be you trying to hard. Finally, some time after you interview you may receive a phone call or email in regards to your status come match day. Take this information as you want but know that not all programs are truthful in regards to where you may fall on their rank list and it isn’t in their interest to be. When you make your rank list, rank where you want to be, not where you’ve gotten the best feedback. If you get a negative response from a program, be ok with this. This means that they saw something in you that they didn’t see working out in the long run. Look at this as a good thing, and don’t react by calling demanding an answer as to why you won’t be ranked or sending a scathing email expressing your feelings of how mad you are. This does not only hurt you there but these messages can and will be forwarded to other programs you are interested in. Know that the interviewing process is an exciting time. You are taking part in choosing where you spend an additional three or four years to train in the coolest specialty in medicine. Know that if you are interviewing at a program that means that they are seriously considering you and want to hear from you why you should be in their program. +

page 33


The Fast Track

Fall 2014

The View from the Bottom: A look at how things really were and are.....

W

By Brian Lehnhof, DO PGY-2 @ Kent Hospital

e are in the middle of an interesting time of year. People are beginning to get comfortable in their new roles. Interesting cases have been experienced and exciting procedures are getting done. It is also about this time that many of us begin to contemplate some common themes. Some of us may wonder why people trust us with their lives. Others may wonder if they will make it through their shift. Some may wonder how they have gotten as far as they have. Finally, some may even wonder why we chose to go into emergency medicine. Internship and emergency medicine in general can be stressful, humbling and demanding. Consequently, thoughts such as these are very common. I believe that these introspections can be a very healthy thing. In these moments we are forced to dig deep and seek the inspiration that has driven us to where we are. I often think back to simpler times in my life and bask in the naïve bliss in which I once resided. As a new college freshman on the north shore of Oahu, my free time involved walking from my dorm room down to Kamehameha Highway where I would hitchhike to one of the nearby surf breaks. Life was simple and life was easy. This is not the case anymore; my life, like all of yours, has become very complex and intense. We live an intense life. The nature of our job requirements, as well as the resulting impact it has on our social lives, can generate significant stress. If you are like me, you probably have lots of friends who have taken far different career paths and are now living far different lives. We have chosen a path that puts us in situations that most people cannot fathom. The stress we encounter in our daily lives is something unique to our profession. I was again reminded of the unique role we serve as I rode on a stretcher doing chest compression while being wheeled down through ED from the CT scanner. Huge expectations are placed upon us. People arrive in the state of greatest need that they may have ever or will ever be in and we have to be prepared to act.

page 34

“You are your own first patient,” is a phrase that has stuck with me. In my youth, I was in a lifeguard certification class where it was unmistakably taught that a rescuer is of no good if they become the next victim. Simply put, you cannot help someone if you cannot care


Fall 2014

The Fast Track for yourself. We cannot control or avoid the stress that comes into our lives. In fact, our profession throws us into the deep end of the metaphorical pool of stressful scenarios, a pool that we begged to be in.. Now that we are here, we have to find a way to keep our heads above water. I would like to offer some suggestions that I have learned as I have been struggling to stay metaphorically afloat.

First, it is crucial to keep a reference frame of what is going on. Too often we get so deep in a situation that we lose efficiency. This is the very reason that the leader of code stays at the foot of bed. If you lose sight of the big picture, you are of little value to the team and ultimately the patient. In practicality, this means we may need to stop and take a breath when things getting crazy. Take a few seconds to review what demands are upon you, review the resources you have to work with and rapidly prioritize a plan of attack. Doing this will allow you to approach the situation with a clear mind. Second, document your experiences. I acknowledge, I am using the very “D� word that produces a lot of the stress in our lives. The last thing anyone wants to do is more documentation, however, I have found fulfillment when I take time to recount some of the events of my day by writing in a journal of some sort. This helps to put things in a new context that I did not comprehend while it was occurring in real time. Writing about experiences unveils the feelings and emotions that were constrained during the intense event. In doing this I have been able to better identify the significance of my efforts that at the time seems trivial or inconsequential. If the outcome was unfavorable, I am able to move forward with increased understanding and preparation for the next time. Third, when you start to feel like your efforts do not make a difference, you have a clear indication that you are not connecting with your patients well enough. Take a few minutes to try to befriend a patient. Preferably pick one that is nice, clean and not vomiting. As you take some time to listen to them beyond their HPI, you will make a connection with them and in so doing, you will be reminded of the difference you are truly making in their life. I guarantee this will make a difference in the way you see your role. Finally, take time to do something that matters to you. Find something that you love and make time to enjoy it. You probably already know what this is for you, but it has probably gotten pushed deep into the back of the priority closet. Sacrificing some seemingly precious time to renew your spirit will yield far more than it costs. When I reflect back to the simpler times in life it can be very appealing to go back to a life void of responsibility. However, the degree of reward parallels the effort requirement. Consequently, the problem with a simple life is that it yields simple and quickly fleeting rewards. Nothing has drained me physically, mentally and emotionally in my life as caring for a very sick patient. In like manner, there are few things that can provide the same reward and fulfillment. This is the prize we need to keep our tanks filled with the fuel to press forward each day. I encourage each of us to remember that we are our own patient. Take care of yourself because people need you and what you do makes a difference.

page 35


The Fast Track

Fall 2014

Residency Spotlight

St. Joseph Regional Medical Center – Paterson, NJ –

Size: 24 EM residents (6 per year) Total ED visits per year: 163,000, #3 in volume nationwide last year Hospital size: Level II trauma with 700 beds page 36


What do you do outside the hospital?

What is unique about your program? SJRMC is located in an area of vast diversity. The combination of underserved and immigrants allows us the ability to treat diseases that you wouldn’t necessarily expect to see, or treat diseases with interesting presentations due to the fact that the patients have not received treatment early in the course of the disease or have had no follow up. We see many critically ill patients allowing us abundant opportunities to manage the very sick and perfect a multitude of procedures. Our pod system is set up with a 1:1 ratio between attending and resident guaranteeing all procedures within your pod belong to you as does the attention of the attending as you need it. Being located right next to a major freeway and in an underserved area we see a nice balance of penetrating and blunt trauma that keeps you on your toes. Our Pediatric EM department sees a wide variety and gives us the confidence to manage the sickest kiddos.

Fall 2014

The Fast Track

What to do outside the hospital: Literally everything! NYC is located a mere 30 minutes from the hospital. We are surrounded by world class dining, entertainment, and sporting events. If outdoors is more your thing, don’t worry we have plenty of that too. NJ is called the garden state for a reason. We have beautiful parks, hiking, biking, white water rafting, fishing, skiing, and just about anything else you can think of within an hour’s drive and often less. If you like to travel we have 3 major airports and a cruise port at your service.

What three words describe your residency? 1. Thrilling 2. Supportive 3. Scholarly

page 37


The Fast Track

Toxic Algal Blooms

Fall 2014

Alexandra Murray, OMS-IV @ OU-HCOM On August 2, 2014 the City of Toledo, Ohio issued a “Do Not Drink” advisory for users of the City’s public water system. This advisory was issued as a result of high levels of an algal toxin detected in the drinking water reservoir supplied by Lake Erie. On August 3, 2014 Ohio Governor John Kasich issued a state of emergency concerning the toxic algae bloom. The drinking water advisory was subsequently lifted on August 4, 2014 as levels of the toxin had subsided to an acceptable level for drinking water.1,2 Lake Erie has yearly algal blooms that generally occur in the late summer and early fall without significant effect on the water supply.3,4 The 2014 algal bloom created the Toledo water crisis due to the location of the bloom at the mouth of the Maumee River where Toledo’s water treatment plant is located.1 During the four day Toledo water crisis local emergency departments saw a dramatic increase in people seeking evaluation after possible exposure; however, no one needed admission or had any life threatening sequelae .2 Although no one suffered any serious harm from the algal toxins during the Toledo water crisis, this incident demonstrates the vital role that Emergency Medicine plays in disaster relief. This event raised awareness of the potential danger of algal toxins and reminds Emergency physicians to be conscious of the clinical signs and symptoms related to cyanobacterial toxicities. Current research suggests that algal blooms may continue to worsen in upcoming years and it is important for Emergency providers to be prepared for the potential crisis that can arise from a contaminated water supply.3 Toxic algal blooms occur every year in freshwater lakes and brackish waters in locations around the world. These blooms occur because of excessive accumulation of microscopic photosynthesizing aquatic organisms (phytoplankton) that produce biotoxins.3-12 Prime locations for these harmful blooms are lakes that are warm, shallow, and have slow moving waters. Land use, agricultural practices, and meteorological conditions that create large deposits of nitrates and phosphorus in the soil contribute to stimulating and exacerbating the blooms. The nitrate and phosphorus runoff in the lakes increases the alkalinity and CO2 in the water, which creates an optimal environment for the algae.3-14 The type of phytoplankton that dominates in Lake Erie is cyanobacteria, commonly known as blue-green algae. Because of the large area of farm country in Northwest Ohio, Lake Erie has long been host to recurrent algal blooms.3-14

page 38

The toxins produced by the cyanobacteria - rather than the cyanobacteria itself - are responsible for the health problems associated with algal blooms. These toxins are resistant to extreme temperatures, which is why boiling and other treatments are ineffective in preventing illness.5-10. Cyanobacterial toxins are grouped according to the physiological systems, organs, tissues, or cells which are primarily affected.5-10 They include: • Hepatotoxins: Microcystins and Pentapeptide Nodularins inhibit protein phosphatases, cause changes in membrane integrity and conductance, cause major liver damage, and are tumor promoters.5-8 • Neurotoxins: Anatoxin-a and Homoanatoxin-a are postsynaptic, cholinergic neuromuscular blocking agents. Saxitoxins block sodium channels.5-8 • Cytotoxins: Cylindrospermopsin is an inhibitor of protein synthesis which causes widespread necrotic injury in mammals (liver, kidneys, lungs, spleen, intestine). It is also enotoxic and can cause chromosome loss and DNA strand breakage.5-8 • Dermatotoxins: Aplysiatoxin, Debromoaplysiatoxin, and Lyngbyatoxin cause skin irritation and are tumor promoters.5-8 • Gastrointestinal toxins: Lipopolysaccharide endotoxins (LPS) may contribute to inflammatory and gastrointestinal incidents.5-8 Emergency Medicine physicians should be aware that cyanobacterial toxins can cause both acute and chronic illness and people can be exposed to these toxins in a number of different ways. Microcystin LR is the toxin most often implicated in cyanobacterial toxicosis and its major route of exposure is through oral ingestion. Patients should be asked whether they have drank water from or swam in a lake reservoir containing algae, worn clothes washed in contaminated water, taken any supplements made from fish products, ate fish from a contaminated lake, or received dialysis from an improperly


treated water source.1,5-10 Pilotto et al. (1997) found that people exposed to greater than 1 hour of recreational water activity in a lake contaminated with high levels of cyanobacteria were more likely to report symptoms.13 Further research by Stewart et al. (2006) showed that people who used personal watercrafts on lakes contaminated with high cyanobacteria levels were 2.1 times more likely to report symptoms. Of the symptoms reported in this study, respiratory symptoms were more common, suggesting that watercrafts may aerosolize the toxins and facilitate infection via inhalation.14,15

Fall 2014

The Fast Track

When evaluating patients with possible cyanotoxin exposure in the ED it is important to remember that most symptoms are nonspecific and can occur minutes to days after exposure.5-10 Common symptoms that have been reported include fever, dyspnea, headaches, dizziness, numbness, tingling, muscle and joint pain, blisters, stomach cramps, diarrhea, vomiting, mouth ulcers and allergic reactions. However, as emergency physicians it is crucial to be able to pick up severe cases of cyanotoxin toxicity that can manifest as liver failure, seizures, kidney failure, or respiratory arrest.5-10 In a report by Giannuzzi et al (2011) there was a case of a young man who was accidentally immersed in a heavily concentrated microcystin algal bloom and four hours later he developed nausea, abdominal pain, dyspnea and fever. Three days later the patient was noted to be in respiratory distress with acute renal failure. He required ventilatory support and was treatment in the ICU for atypical pneumonia. One week after his initial exposure the patient was noted to have hepatotoxicosis with serious liver damage. Fortunately, the patient was able to completely recover over the course of 20 days.15 This case serves to illustrate the rapidity and potential severity of microcystin toxicosis as well as demonstrate the need for a high index of suspicion when dealing with patients who have recently been exposed to algal blooms. Toxic algal blooms can occur in any freshwater lake around the world. Recognizing the signs and symptoms of cyanobacterial poisoning is important for all ED physicians because several cities receive their water supply from freshwater lakes. There are currently no cyanobacterial antitoxins available and it is recommended that people suffering from toxin related symptoms receive supportive therapy.5,6 With the worsening of blooms over the past few years, more research is needed to determine the best way to detect and treat these toxin related sequelae. There has already been a link discovered between cyanobacterial toxins and certain late stage diseases, such as hepatocellular carcinoma and other neoplasms, but research is still ongoing as to better understand this relationship.17,18 nated with high cyanobacteria levels were 2.1 times more likely to report symptoms. Of the symptoms reported in this study, respiratory symptoms were more common, suggesting that watercrafts may aerosolize the toxins and facilitate infection via inhalation.14,15 When evaluating patients with possible cyanotoxin exposure in the ED it is important to remember that most symptoms are nonspecific and can occur minutes to days after exposure.5-10 Common symptoms that have been reported include fever, dyspnea, headaches, dizziness, numbness, tingling, muscle and joint pain, blisters, stomach cramps, diarrhea, vomiting, mouth ulcers and allergic reactions. However, as emergency physicians it is crucial to be able to pick up severe cases of cyanotoxin toxicity that can manifest as liver failure, seizures, kidney failure, or respiratory arrest.5-10 In a report by Giannuzzi et al (2011) there was a case of a young man who was accidentally immersed in a heavily concentrated microcystin algal bloom and four hours later he developed nausea, abdominal pain, dyspnea and fever. Three days later the patient was noted to be in respiratory distress with acute renal failure. He required ventilatory support and was treatment in the ICU for atypical pneumonia. One week after his initial exposure the patient was noted to have hepatotoxicosis with serious liver damage. Fortunately, the patient was able to completely recover over the course of 20 days.15 This case serves to illustrate the rapidity and potential severity of microcystin toxicosis as well as demonstrate the need for a high index of suspicion when dealing with patients who have recently been exposed to algal blooms. Toxic algal blooms can occur in any freshwater lake around the world. Recognizing the signs and symptoms of cyanobacterial poisoning is important for all ED physicians because several cities receive their water supply from freshwater lakes. There are currently no cyanobacterial antitoxins available and it is recommended that people suffering from toxin related symptoms receive supportive therapy.5,6 With the worsening of blooms over the past few years, more research is needed to determine the best way to detect and treat these toxin related sequelae. There has already been a link discovered between cyanobacterial toxins and certain late stage diseases, such as hepatocellular carcinoma and other neoplasms, but research is still ongoing as to better understand this relationship.17,18

page 39


The Fast Track

References Fall 2014

p.10 - STEMI ALERT! by Drew Kalnow, DO Kurz M, Mattu A, Brady W. Acute Coronary Syndrome. In: Marx J, Hockberger R, Walls R. Rosen’s Emergency Medicine 8ed. Philadelphia, PA: Elsevier Saunders; 2014: 998-1033. Sharkey S, Lesser J, Maron B. Takotsubo (Stress) Cardiomyopathy. Circulation [serial online]. 2011; 124: e460-e462. Available at: http://circ.ahajournals.org/content/124/18/e460.full Tomich E. Takotsubo Cardiomyopathy. Medscape Web Site. 2012. Available at: http://emedicine.medscape.com/article/1513631-overview#aw2aab6b2b2aa. Accessed September 1, 2014. Weber J. Acute Coronary Syndromes: Management of Myocardial Infarction and Unstable Angina. In Ma O, Cline D. Emergency Medicine Manual 6ed. New York, NY: McGraw-Hill: 129-136. p.16 - Medical Apps for Medical Students by Nicholas Pettit iTunes Store: https:\\www.apple.com/itunes/ 2. Dolan, B. Top five medical apps at Harvard Medical School. Mobile Health News. 2011. Available at http://mobihealthnews.com/10745/top-five-medical-apps-at-harvard-medical-school. Accessed August 10, 2014. 3. Husain, I. The best medical apps for your new iPhone. iMedicalApps. Available at http://www.imedicalapps.com/2014/01/medical-apps-iphone/3/. Accessed August 10, 2014. 4. Buijink AWG, Visser BJ, Marshall L. Medical Apps for Smartphones: Lack of Evidence undermines quality and safety. Evid Based Med. 2014; 19(3):8587. p.18 - Hyperlipasemia associated with acute, colonic diverticulitis by Dhimitri Nikolla and Brandon Kramer, DO Jacobs D. “Diverticulitis.” N Engl J Med. 2007;357:2057-66. Whitcomb D. “Acute Pancreatitis.” N Engl J Med. 2006; 354:2142-50. Klingensmith, Mary E. The Washington Manual of Surgery, Fifth Edition. Philadelphia, PA: Wolters Kluwer Health/Lippincott Williams & Wilkins; 2008: 239. Manikkan A. “Hyperlipasemia in Diabetic Ketoacidosis.” Clinical Diabetes. 2013; 31 (1): 31-2. Sinha S, Khan H, Timms PM, et al. “Pancreatic-type hyperamylasemia and hyperlipasemia secondary to ruptured ovarian cyst: a case report and review of the literature.” J Emerg Med. 2010; 38 (4):463-6. Dylewski ML, Prelack K, Keaney T, et al. “Asymptomatic hyperamylasemia and hyperlipasemia in pediatric patients with toxic epidermal necrolysis.” J Burn Care Res. 2010; 31 (2):292-6. Lattanzi L, Casamassima F, Brunetto M, et al. “Asymptomatic hyperamylasemia and hyperlipasemia associated with aripiprazole.” J Clin Psychopharmacol. 2009; 29 (5):504-6. Lionetti E, Francavilla R, Leonardi S, et al. “Two rare cases of benign hyperlipasemia in children.” World J Clin Cases 2014; 2(1): 16-19. p.22 - Dealing with Death as a Medical Student by James Chapman Marrow, Jonathan. “Telling Relatives That a Family Member Has Died Suddenly.” Postgraduate Medical Journal 72.849 (1996): 413-18. Boog, Kathryn M., and Claire Y. Tester. Palliative Care: A Practical Guide for the Health Professional: Finding Meaning and Purpose in Life and Death. Edinburgh: Churchill Livingstone Elsevier, 2008. p. 26 - Brain Doping by Giles Gifford Herman L, Shtayermman O, Aksnes B, Anzalone M, Cormerias A, Liodice C. The use of prescription stimulants to enhance academic performance among college students in health care programs. Journal of Physician Assistant Education 2011;22(4):15-22. McCabe S,West B, Teter C, Boyd C. “ Trends in Medical Use, Diversion, and None Medical Use of Prescription Medications Among College Students From 2003 to 2013: Connecting the Dots”. Addictive Behaviors, An International Journal; Volume 39. Issue 7 July 2014, Pages 1176-1182 Schick, David “Adderall does not teach you calculus” www.usatoday.com June 20th 2013

page 40

New Health Guide “How Long Does Adderall Stay in your System” www. newhealthguide.org/ Sept 11th 2014 http://www.wikihow.com/Get-an-Adderall-Prescription Crawford C. “ AAFP Joins Diverse Coalition That Aims to Prevent ADHD Medication Misuse” American Academy of Family Physicians. http://www. aafp.org September 02, 2014 p.28 - Pimpology 101 by Danielle Turrin, DO, MS http://emedicine.medscape.com/article/156951-overview http://lifeinthefastlane.com/ecg-library/basics/pericarditis/ p.38 - Toxic Algal Blooms by Alexandra Murray Microcystin Event Preliminary Summary. City of Toledo Department of Public Utilities. August 4, 2014. Available at: http://www.toledoblade.com/ attachment/2014/08/04/72-page-preliminary-study-from-the-City-of-Toledo-on-water-crisis.pdf Harris-Taylor, M. (2014, August 3). Toledo area hospitals see rise in visitors to emergency rooms. The Toledo Blade. Retrieved April 21, 2014, from http:// www.toledoblade.com/Medical/2014/08/03/Toledo-area-hospitals-see-risein-visitors-to-emergency-rooms.html Michalak AM et al. Record-setting algal bloom in Lake Erie caused by agricultural and meteorological trends consistent with expected future conditions. Proc Natl Acad Sci U S A. 2013 Apr 16;110(16):6448-52. Verspagen JM et al. Rising CO2 Levels Will Intensify Phytoplankton Blooms in Eutrophic and Hypertrophic Lakes. PLoS One. 2014 Aug 13;9(8):e104325. Bartram J, Chorus I. Toxic Cyanobacteria in Water: A Guide to their Public Health Consequences, Monitoring and Management. CRC Press; 2002. Cyanobacterial Toxins: Microcystin-LR in Drinking Water. In: Guidelines for drinking-water quality, 2nd ed. Addendum to Vol. 2. Health criteria and other supporting information. World Health Organization. Geneva; 1998. Codd GA, Morrison LF, Metcalf JS. Cyanobacterial toxins: risk management for health protection. Toxicol Appl Pharmacol. 2005 Mar 15;203(3):264-72. Cyanobacteria and Cyanotoxins: Information for Drinking Water Systems. United States Emergency Protection Agency, Office of Water. July 2012. Available at: http://www2.epa.gov/nutrient-policy-data/cyanobacterialharmful-algal-blooms-cyanohabs Paerl HW, Fulton RS, Moisander PH, Dyble J. Harmful freshwater algal blooms, with an emphasis on cyanobacteria. ScientificWorldJournal. 2001 Apr 4;1:76-113. Hilborn ED et al. Algal bloom-associated disease outbreaks among users of freshwater lakes - United States, 2009-2010. MMWR Morb Mortal Wkly Rep. 2014 Jan 10;63(1):11-5. Paerl HW et al. Controlling harmful cyanobacterial blooms in a hyper-eutrophic lake (Lake Taihu, China): the need for a dual nutrient (N & P) management strategy. Water Res. 2011 Feb;45(5):1973-83. Muhid P, Davis TW, Bunn SE, Burford MA. Effects of inorganic nutrients in recycled water on freshwater phytoplankton biomass and composition. Water Res. 2013 Jan 1;47(1):384-94. Pilotto et al. Health effects of exposure to cyanobacteria (blue-green algae) during recreational water-related activities. Aust. N. Z. J. Public Health. 1997; 21: 562-566. Stewart I et al. Epidemiology of recreational exposure to freshwater cyanobacteria--an international prospective cohort study. BMC Public Health. 2006 Apr 11;6:93. Baker LC et al. Recreational exposure to microcystins during algal blooms in two California lakes. Toxicon. 2010 May;55(5):909-21. Giannuzzi L, Sedan D, Echenique R, Andrinolo D. An acute case of intoxication with cyanobacteria and cyanotoxins in recreational water in Salto Grande Dam, Argentina. Mar Drugs. 2011;9(11):2164-75. Zhang X et al. Toxic effects of microcystin-LR on the HepG2 cell line under hypoxic and normoxic conditions. J Appl Toxicol. 2013 Oct;33(10):1180-6. Nong Q et al. Involvement of reactive oxygen species in Microcystin-LRinduced cytogenotoxicity. Free Radic Res. 2007 Dec;41(12):1326-37. Errata from Summer 2014 editon: Zachary Barfield @ VCOM should have credited as the photographer for “Snake Bites: A case study and discussion” on pages 20-21.


Emergency Medicine Review with

Elevate your knowlege, rock the inservice exam and blow away the boards! www.roshreview.com

Fall 2014

The Fast Track

page 41


Review Answers

The Fast Track

Fall 2014

Question 1: Answer A. This patient presents with a severe headache unlike prior headaches raising the suspicion for a subarachnoid hemorrhage (SAH). SAH is defined as the extravasation of blood into the subarachnoid space. Approximately 1-4% of patients presenting to the ED for headache will have a SAH. The classic presentation of SAH is a patient presenting with a “thunderclap” headache or “the worst headache of life.” However, it should be suspected in any patient who presents with an acute, severe headache that reaches maximal intensity in a short period of time or is different than prior headaches. Although some patients will present with traumatic SAH, the most dangerous form is SAH related to berry aneurysms. These aneurysms are prone to catastrophic rupture. Many patients who have large berry aneurysm bleeds will report a sentinel headache, which may or may not have prompted a doctor visit. It is vital to diagnose patients presenting with a sentinel bleed as neurosurgical intervention for berry aneurysms can avoid catastrophic events. Noncontrast CT scan of the head should be ordered in any patient with a presentation concerning for SAH. The sensitivity of CT within the first 6 hours of headache onset is extremely high (> 95%) but the sensitivity decreases with time. Overall sensitivity is about 93%. A negative CT scan should be followed by a lumbar puncture (LP). The LP is performed looking for the presence of blood in the CSF or xanthochromia (the yellowish pigmentation of CSF secondary to hemoglobin metabolism). Xanthochromia takes approximately 12 hours to develop. Non-contrast head CT scan (B) alone is highly sensitive for the diagnosis of SAH if performed within the first 6 hours. However, because SAH is a life-threatening disease if missed, current guidelines recommend against using non-contrast head CT to rule out the diagnosis. An MRI of the brain (C) can be helpful in detecting berry aneurysms as the cause of SAH but the workup should not be delayed when SAH is suspected. Symptomatic treatment (D) should be given to all headache patients regardless of etiology. However, response to treatment does not rule out dangerous pathology. References Kwiatkowski T, Alagappan K: Headache, in Marx JA, Hockberger RS, Walls RM, et al (eds): Rosen’s Emergency Medicine: Concepts and Clinical Practice, ed 7. St. Louis, Mosby, Inc., 2010, (Ch) 101: p 1356-1366. Question 2: Answer: C. This patient presents with a classic story for an epidural hematoma confirmed by imaging and requires immediate evaluation for prompt surgical hematoma evacuation. Epidural hematoma results from trauma leading to rupture of the middle meningeal artery. Classically, patients report a brief loss of consciousness followed by a lucid period and then a precipitous decline. Non-contrast head CT (NCHCT) scan is diagnostic for this disease. A NCHCT will show a biconvex or lens shaped hyperdense area in the temporal region of the side of trauma in the epidural space. Depending on the size of the hematoma, there may also be midline shift and compression of the ventricles. As the hematoma continues to expand, it can cause uncal herniation leading to brain compromise and respiratory arrest. In an obtunded patient, the physical examination may reveal a unilateral dilated and unresponsive pupil (aka “blown pupil) on the side of the hematoma. Emergency management of an epidural hematoma should focus on airway protection if necessary and emergent neurosurgical consultation for hematoma evacuation. In patients with signs of increased intracranial pressure or impending herniation, hyperventilation should be initiated followed by osmotic agents like mannitol.

page 42

Stable patients with traumatic subdural or subarachnoid hematomas may be admitted for serial NCHCT (A) after neurosurgical evaluation. These intracranial hematomas may not increase in size and may not require surgical intervention. However, this does not apply to epidural hematomas. In the absence of an intracranial hematoma, the patient’s symptoms may be consistent with a concussion, which would require outpatient neurology follow up (B). MRI of the brain (D) is not necessary in epidural hematoma


as the NCHCT is diagnostic. References Kwiatkowski T, Alagappan K: Headache, in Marx JA, Hockberger RS, Walls RM, et al (eds): Rosen’s Emergency Medicine: Concepts and Clinical Practice, ed 7. St. Louis, Mosby, Inc., 2010, (Ch) 101: p 1356-1366.

Fall 2014

The Fast Track

Question 3: Answer: D. Therapeutic hypothermia has become the standard of care in the care of postarrest patients who achieve return of spontaneous circulation but remain comatose. The original trials showed significant survival benefit in patients whose initial rhythm was either ventricular fibrillation or pulseless ventricular tachycardia. Additional observational data has suggested a modest benefit in the other rhythms and the American Heart Association now recommends therapeutic hypothermia in all unresponsive post-cardiac arrest patients. In the first hours after cardiac arrest, the core temperature of the body is lowered and maintained there for 12 to 24 hours. Recent data suggests that the most important element in post-cardiac arrest care is avoidance of fever. Hyperkalemia (A) may lead to cardiac arrest. In most cases the rhythms associated with hyperkalemia are either asystole or pulseless electrical activity, which appear to have slightly worse outcomes when compared with ventricular fibrillation or ventricular tachycardia in therapeutic hypothermia. Pulmonary embolism (B) may lead to cardiac arrest with significant proximal clot or overwhelming clot burden. However, outcome data of therapeutic hypothermia looks at primary rhythm in arrest, and most commonly pulmonary embolism is associated with pulseless electrical activity or asystole. Patients found in pulseless electrical activity (C) may derive some survival benefit from therapeutic hypothermia but the benefit is not as strong as ventricular fibrillation or ventricular tachycardia. References http://www.uptodate.com/contents/post-cardiac-arrest-management-in-adults?source=search_result&s earch=therapeutic+hypothermia&selectedTitle=1~32#H9724176 Question 4: Answer: C. Systemic inflammatory response syndrome (SIRS) is the systemic inflammatory response syndrome occurring most commonly in response to an infection. SIRS plus a source of infection is the definition of sepsis. Sepsis is an increasingly common cause of ED visits and the tenth leading cause of death in the US. In response to the infectious insult, the host’s body activates a number of inflammatory cascades as part of the host response. As patients become sicker, severe sepsis develops with the presence of organ dysfunction in the setting of an infection. The respiratory rate of 22 above fulfills one of the SIRS criteria. The heart rate of 86 (A) and temperature of 39.7°C (D) do not meet criteria for SIRS. Lactic acid (B) has become an important laboratory value in the management of septic patients. Lactic acids greater than 4 are associated with worsened outcomes. Aggressive resuscitation in the early stages of sepsis includes large volumes of intravenous fluid and early antibiotics. A goal of resuscitation is lactate clearance.

Reference: Shapiro NI, Zimmer GD, Barkin AZ: Sepsis Syndromes, in Marx JA, Hockberger RS, Walls RM, et al (eds): Rosen’s Emergency Medicine: Concepts and Clinical Practice, ed 8. St. Louis, Mosby, Inc., 2013, (Ch) 138: pp 1864-1866.

page 43


The Fast Track

Fall 2014

ACOEP Resident and Student Chapter 142 East Ontario Street Suite 1500 Chicago, Illinois 60611 Phone: 312.587.3709 page Fax: 312.587.9951 44 E-mail: fasttrack@acoep.org

Follow us on

@ ACOEPSC @ ACOEPRC @ facebook.com/acoepsc @ facebook.com/acoeprc


Issuu converts static files into: digital portfolios, online yearbooks, online catalogs, digital photo albums and more. Sign up and create your flipbook.